1 Introduction and preliminaries

Very recently, we studied in [1] the existence and uniqueness of fixed points for self-operators defined in a metric space and belonging to the class of \((\alpha,\psi)\)-type contraction mappings (see [25] for some works in this direction). We proved that the class of α-ψ-type contractions includes large classes of contraction-type operators, whose fixed points can be obtained by means of the Picard iteration. The aim of this paper is to extend the obtained results in [1] to self-operators defined in a b-metric space.

We start by recalling the following definition.

Definition 1.1

([6])

Let X be a nonempty set. A mapping \(d: X\times X \to[0,\infty)\) is called b-metric if there exists a real number \(b\geq1\) such that for every \(x,y,z\in X\), we have

  1. (i)

    \(d(x,y)=0\) if and only if \(x=y\);

  2. (ii)

    \(d(x,y)=d(y,x)\);

  3. (iii)

    \(d(x,z)\leq b[d(x,y)+d(y,z)]\).

In this case, the pair \((X,d)\) is called a b-metric space.

There exist many examples in the literature (see [68]) showing that the class of b-metrics is effectively larger than that of metric spaces.

The notions of convergence, compactness, closedness and completeness in b-metric spaces are given in the same way as in metric spaces. For works on fixed point theory in b-metric spaces, we refer to [912] and the references therein.

Definition 1.2

([13])

Let \(\psi:[0,\infty)\to[0,\infty)\) be a given function. We say that ψ is a comparison function if it is increasing and \(\psi^{n}(t)\to 0\), \(n\to\infty\), for any \(t\geq0\), where \(\psi^{n}\) is the nth iterate of ψ.

In [13, 14], several results regarding comparison functions can be found. Among these we recall the following.

Lemma 1.3

If \(\psi:[0,\infty)\to[0,\infty)\) is a comparison function, then

  1. (i)

    each iterate \(\psi^{k}\) of ψ, \(k\geq1\), is also a comparison function;

  2. (ii)

    ψ is continuous at zero;

  3. (iii)

    \(\psi(t)< t\) for any \(t >0\);

  4. (iv)

    \(\psi(0)=0\).

The following concept was introduced in [15].

Definition 1.4

Let \(b\geq1\) be a real number. A mapping \(\psi:[0,\infty)\to[0,\infty )\) is called a b-comparison function if

  1. (i)

    ψ is monotone increasing;

  2. (ii)

    there exist \(k_{0}\in\mathbb{N}\), \(a\in(0,1)\) and a convergent series of nonnegative terms \(\sum_{k=1}^{\infty}v_{k}\) such that

    $$b^{k+1}\psi^{k+1}(t)\leq a b^{k} \psi^{k}(t)+v_{k} $$

    for \(k\geq k_{0}\) and any \(t\geq0\).

The following lemma has been proved.

Lemma 1.5

([15, 16])

Let \(\psi:[0,\infty)\to[0,\infty)\) be a b-comparison function. Then

  1. (i)

    the series \(\sum_{k=0}^{\infty}b^{k}\psi ^{k}(t)\) converges for any \(t\geq0\);

  2. (ii)

    the function \(s_{b}:[0,\infty)\to[0,\infty)\) defined by

    $$s_{b}(t)=\sum_{k=0}^{\infty}b^{k}\psi^{k}(t), \quad t\geq0 $$

    is increasing and continuous at 0.

Lemma 1.6

([17])

Any b-comparison function is a comparison function.

Throughout this paper, for \(b\geq1\), we denote by \(\Psi_{b}\) the set of b-comparison functions.

Definition 1.7

Let \((X,d)\) be a b-metric space with constant \(b\geq1\), and let \(T: X\to X\) be a given mapping. We say that T is an α-ψ contraction if there exist a b-comparison function \(\psi\in\Psi_{b}\) and a function \(\alpha:X\times X\to\mathbb{R}\) such that

$$ \alpha(x,y)d(Tx,Ty)\leq\psi\bigl(d(x,y)\bigr) \quad \text{for all } x,y\in X. $$
(1.1)

2 Main results

Let \(T: X\to X\) be a given mapping. We denote by \(\operatorname{Fix}(T)\) the set of its fixed points; that is,

$$\operatorname{Fix}(T)=\{x\in X: x=Tx\}. $$

For \(b\geq1\) and \(\psi\in\Psi_{b}\), let \(\Sigma_{\psi}^{b}\) be the set defined by

$$\Sigma_{\psi}^{b}=\bigl\{ \sigma\in(0,\infty): \sigma\psi\in \Psi_{b}\bigr\} . $$

We have the following result.

Proposition 2.1

Let \((X,d)\) be a b-metric space with constant \(b\geq1\), and let \(T: X\to X\) be a given mapping. Suppose that there exist \(\alpha: X\times X \to\mathbb{R}\) and \(\psi\in\Psi_{b}\) such that T is an α-ψ contraction. Suppose that there exists \(\sigma\in\Sigma_{\psi}^{b}\) and for some positive integer p, there exists a finite sequence \(\{\xi_{i}\}_{i=0}^{p}\subset X\) such that

$$ \xi_{0}=x_{0}, \qquad \xi_{p}=Tx_{0}, \qquad \alpha\bigl(T^{n}\xi_{i},T^{n} \xi_{i+1}\bigr)\geq\sigma^{-1}, \quad n\in\mathbb {N}, i=0, \ldots,p-1, x_{0}\in X. $$
(2.1)

Then \(\{T^{n}x_{0}\}\) is a Cauchy sequence in \((X,d)\).

Proof

Let \(\varphi=\sigma\psi\). By the definition of \(\Sigma_{\psi}^{b}\), we have \(\varphi\in\Psi_{b}\). Let \(\{\xi_{i}\}_{i=0}^{p}\) be a finite sequence in X satisfying (2.1). Consider the sequence \(\{x_{n}\}_{n\in\mathbb{N}}\) in X defined by \(x_{n+1}=Tx_{n}\), \(n\in\mathbb{N}\). We claim that

$$ d\bigl(T^{r}\xi_{i}, T^{r} \xi_{i+1}\bigr)\leq\varphi^{r}\bigl(d(\xi_{i}, \xi_{i+1})\bigr),\quad r\in\mathbb{N}, i=0,\ldots,p-1. $$
(2.2)

Let \(i\in\{0,1,\ldots,p-1\}\). From (2.1), we have

$$\sigma^{-1}d(T\xi_{i}, T\xi_{i+1})\leq\alpha( \xi_{i},\xi_{i+1})d(T\xi_{i}, T\xi_{i+1}) \leq\psi\bigl(d(\xi_{i},\xi_{i+1})\bigr), $$

which implies that

$$ d(T\xi_{i}, T\xi_{i+1})\leq\varphi\bigl(d( \xi_{i},\xi_{i+1})\bigr). $$
(2.3)

Again, we have

$$\sigma^{-1}d\bigl(T^{2}\xi_{i}, T^{2} \xi_{i+1}\bigr) \leq\alpha(T\xi_{i},T\xi_{i+1})d \bigl(T(T\xi_{i}), T(T\xi_{i+1})\bigr) \leq\psi\bigl(d(T \xi_{i},T\xi_{i+1})\bigr), $$

which implies that

$$ d\bigl(T^{2}\xi_{i}, T^{2} \xi_{i+1}\bigr)\leq\varphi\bigl(d(T\xi_{i},T \xi_{i+1})\bigr). $$
(2.4)

Since φ is an increasing function (from Lemma 1.6), from (2.3) and (2.4), we obtain

$$d\bigl(T^{2}\xi_{i}, T^{2}\xi_{i+1} \bigr)\leq\varphi^{2}\bigl(d(\xi_{i},\xi_{i+1}) \bigr). $$

Continuing this process, by induction we obtain (2.2).

Now, using the property (iii) of a b-metric and (2.2), for every \(n\in\mathbb{N}\), we have

$$\begin{aligned} d(x_{n},x_{n+1})&= d\bigl(T^{n}x_{0},T^{n+1}x_{0} \bigr) \\ &\leq b d\bigl(T^{n}\xi_{0},T^{n} \xi_{1}\bigr)+b^{2}d\bigl(T^{n} \xi_{1},T^{n}\xi_{2}\bigr) +\cdots+ b^{p}d\bigl(T^{n}\xi_{p-1},T^{n} \xi_{p}\bigr) \\ &= \sum_{i=0}^{p-1}b^{i+1}d \bigl(T^{n}\xi_{i},T^{n}\xi_{i+1}\bigr) \\ &\leq \sum_{i=0}^{p-1} b^{i+1} \varphi^{n}\bigl(d(\xi_{i},\xi_{i+1})\bigr). \end{aligned}$$

Thus we proved that

$$d(x_{n},x_{n+1})\leq\sum_{i=0}^{p-1} b^{i+1}\varphi^{n}\bigl(d(\xi_{i},\xi _{i+1})\bigr),\quad n\in\mathbb{N}, $$

which implies that for \(q\geq1\),

$$\begin{aligned} d(x_{n},x_{n+q}) &\leq \sum_{j=n}^{n+q-1} b^{j-n+1}d(x_{j},x_{j+1}) \\ &\leq \sum_{j=n}^{n+q-1} b^{j-n+1}\sum _{i=0}^{p-1} b^{i+1}\varphi ^{j}\bigl(d(\xi_{i},\xi_{i+1})\bigr) \\ &= \frac{1}{b^{n-1}}\sum_{i=0}^{p-1}b^{i+1} \sum_{j=n}^{n+q-1} b^{j} \varphi^{j}\bigl(d(\xi_{i},\xi_{i+1})\bigr) \\ &\leq\frac{1}{b^{n-1}}\sum_{i=0}^{p-1}b^{i+1} \sum_{j=n}^{\infty} b^{j} \varphi^{j}\bigl(d(\xi_{i},\xi_{i+1})\bigr). \end{aligned}$$

Since \(b\geq1\), using Lemma 1.5(i), we obtain

$$\frac{1}{b^{n-1}}\sum_{i=0}^{p-1}b^{i+1} \sum_{j=n}^{\infty} b^{j}\varphi ^{j}\bigl(d(\xi_{i},\xi_{i+1})\bigr)\to0\quad \text{as } n \to\infty. $$

This proves that \(\{x_{n}\}\) is a Cauchy sequence in the b-metric space \((X,d)\). □

Our first main result is the following fixed point theorem which requires the continuity of the mapping T.

Theorem 2.2

Let \((X,d)\) be a complete b-metric space with constant \(b\geq1\), and let \(T: X\to X\) be a given mapping. Suppose that there exist \(\alpha: X\times X \to\mathbb{R}\) and \(\psi \in\Psi_{b}\) such that T is an α-ψ contraction. Suppose also that (2.1) is satisfied. Then \(\{T^{n}x_{0}\}\) converges to some \(x^{*}\in X\). Moreover, if T is continuous, then \(x^{*}\in\operatorname{Fix}(T)\).

Proof

From Proposition 2.1, we know that \(\{ T^{n}x_{0}\}\) is a Cauchy sequence. Since \((X,d)\) is a complete b-metric space, there exists \(x^{*}\in X\) such that

$$\lim_{n\to\infty} d\bigl(T^{n}x_{0},x^{*}\bigr)=0. $$

The continuity of T yields

$$\lim_{n\to\infty} d\bigl(T^{n+1}x_{0},Tx^{*} \bigr)=0. $$

By the uniqueness of the limit, we obtain \(x^{*}=Tx^{*}\), that is, \(x^{*}\in \operatorname{Fix}(T)\). □

In the next theorem, we omit the continuity assumption of T.

Theorem 2.3

Let \((X,d)\) be a complete b-metric space with constant \(b\geq1\), and let \(T: X\to X\) be a given mapping. Suppose that there exist \(\alpha: X\times X \to\mathbb{R}\) and \(\psi \in\Psi_{b}\) such that T is an α-ψ contraction. Suppose also that (2.1) is satisfied. Then \(\{T^{n}x_{0}\}\) converges to some \(x^{*}\in X\). Moreover, if there exists a subsequence \(\{T^{\gamma(n)}x_{0}\}\) of \(\{T^{n}x_{0}\}\) such that

$$\max\bigl\{ \alpha\bigl(T^{\gamma(n)}x_{0},x^{*}\bigr), \alpha \bigl(x^{*},T^{\gamma(n)}x_{0}\bigr)\bigr\} \geq\ell\in(0,\infty),\quad n \textit{ large enough}, $$

then \(x^{*}\in\operatorname{Fix}(T)\).

Proof

From Proposition 2.1 and the completeness of the b-metric space \((X,d)\), we know that \(\{T^{n}x_{0}\}\) converges to some \(x^{*}\in X\).

Suppose now that there exists a subsequence \(\{T^{\gamma(n)}x_{0}\}\) of \(\{T^{n}x_{0}\}\) such that

$$ \max\bigl\{ \alpha\bigl(T^{\gamma(n)}x_{0},x^{*}\bigr), \alpha\bigl(x^{*},T^{\gamma(n)}x_{0}\bigr)\bigr\} \geq\ell\in(0, \infty), \quad n \mbox{ large enough}. $$
(2.5)

Since T is an α-ψ contraction, we have

$$\alpha\bigl(T^{\gamma(n)}x_{0},x^{*}\bigr)d\bigl(T^{\gamma(n)+1}x_{0},Tx^{*} \bigr) \leq\psi\bigl(d\bigl(T^{\gamma(n)}x_{0},x^{*}\bigr)\bigr), \quad n\in\mathbb{N} $$

and

$$\alpha\bigl(x^{*},T^{\gamma(n)}x_{0}\bigr)d\bigl(T^{\gamma(n)+1}x_{0},Tx^{*} \bigr) \leq\psi\bigl(d\bigl(T^{\gamma(n)}x_{0},x^{*}\bigr)\bigr), \quad n\in\mathbb{N}. $$

Thus we have

$$\max\bigl\{ \alpha\bigl(T^{\gamma(n)}x_{0},x^{*}\bigr), \alpha \bigl(x^{*},T^{\gamma(n)}x_{0}\bigr)\bigr\} d\bigl(T^{\gamma(n)+1}x_{0},Tx^{*} \bigr) \leq\psi\bigl(d\bigl(T^{\gamma(n)}x_{0},x^{*}\bigr)\bigr),\quad n\in\mathbb{N}. $$

From (2.5), we get

$$ \ell d\bigl(T^{\gamma(n)+1}x_{0},Tx^{*}\bigr)\leq \psi \bigl(d\bigl(T^{\gamma(n)}x_{0},x^{*}\bigr)\bigr),\quad n \mbox{ large enough}. $$
(2.6)

On the other hand, using the property (iii) of a b-metric, we get

$$ d\bigl(T^{\gamma(n)+1}x_{0},Tx^{*}\bigr)\geq \frac{1}{b} d\bigl(x^{*},Tx^{*}\bigr)-d\bigl(x^{*},T^{\gamma (n)+1}x_{0} \bigr), \quad n\in\mathbb{N}. $$
(2.7)

Now, (2.6) and (2.7) yield

$$\ell \biggl(\frac{1}{b} d\bigl(x^{*},Tx^{*}\bigr)-d\bigl(x^{*},T^{\gamma(n)+1}x_{0} \bigr) \biggr)\leq \psi\bigl(d\bigl(T^{\gamma(n)}x_{0},x^{*}\bigr) \bigr), \quad n \mbox{ large enough}. $$

Letting \(n\to\infty\) in the above inequality, using Lemma 1.6 and Lemma 1.3(ii) and (iv), we obtain

$$0\leq\frac{\ell}{b} d\bigl(x^{*},Tx^{*}\bigr)\leq\psi(0)=0, $$

which implies that \(d(x^{*},Tx^{*})=0\), that is, \(x^{*}\in\operatorname {Fix}(T)\). □

We provide now a sufficient condition for the uniqueness of the fixed point.

Theorem 2.4

Let \((X,d)\) be a b-metric space with constant \(b\geq1\), and let \(T: X\to X\) be a given mapping. Suppose that there exist \(\alpha: X\times X \to\mathbb{R}\) and \(\psi \in\Psi_{b}\) such that T is an α-ψ contraction. Suppose also that

  1. (i)

    \(\operatorname{Fix}(T)\neq\emptyset\);

  2. (ii)

    for every pair \((x,y)\in\operatorname{Fix}(T)\times \operatorname{Fix}(T)\) with \(x\neq y\), if \(\alpha(x,y)<1\), then there exists \(\eta\in\Sigma_{\psi}^{b}\) and for some positive integer q, there is a finite sequence \(\{\zeta_{i}(x,y)\}_{i=0}^{q}\subset X\) such that

    $$\zeta_{0}(x,y)=x, \qquad \zeta_{q}(x,y)=y,\qquad \alpha \bigl(T^{n}\zeta_{i}(x,y),T^{n} \zeta_{i+1}(x,y)\bigr)\geq\eta^{-1} $$

    for \(n\in\mathbb{N}\) and \(i=0,\ldots,q-1\).

Then T has a unique fixed point.

Proof

Let \(\varphi=\eta\psi\in\Psi_{b}\). Suppose that \(u,v\in X\) are two fixed points of T such that \(d(u,v)>0\). We consider two cases.

Case 1: \(\alpha(u,v)\geq1\). Since T is an α-ψ contraction, we have

$$d(u,v)\leq\alpha(u,v)d(Tu,Tv)\leq\psi\bigl(d(u,v)\bigr). $$

On the other hand, from Lemma 1.6 and Lemma 1.3(iii), we have

$$\psi\bigl(d(u,v)\bigr)< d(u,v). $$

The two above inequalities yield a contradiction.

Case 2: \(\alpha(u,v)<1\). By assumption, there exists a finite sequence \(\{\zeta_{i}(u,v)\} _{i=0}^{q}\) in X such that

$$\zeta_{0}(u,v)=u, \qquad \zeta_{q}(u,v)=v,\qquad \alpha \bigl(T^{n}\zeta_{i}(u,v),T^{n} \zeta_{i+1}(u,v)\bigr)\geq\eta^{-1} $$

for \(n\in\mathbb{N}\) and \(i=0,\ldots,q-1\). As in the proof of Proposition 2.1, we can establish that

$$ d\bigl(T^{r}\zeta_{i}(u,v), T^{r} \zeta_{i+1}(u,v)\bigr) \leq\varphi^{r}\bigl(d\bigl( \zeta_{i}(u,v),\zeta_{i+1}(u,v)\bigr)\bigr),\quad r\in \mathbb{N}, i=0,\ldots,q-1. $$
(2.8)

On the other hand, we have

$$\begin{aligned} d(u,v) &= d\bigl(T^{n}u,T^{n}v\bigr) \\ &\leq \sum_{i=0}^{q-1} b^{i+1}d \bigl(T^{n}\zeta_{i}(u,v),T^{n} \zeta_{i+1}(u,v)\bigr) \\ &\leq \sum_{i=0}^{q-1}b^{i+1} \varphi ^{n}\bigl(d\bigl(\zeta_{i}(u,v),\zeta_{i+1}(u,v) \bigr)\bigr)\to0\quad \text{as } n\to\infty \mbox{ (by Lemma 1.6)}. \end{aligned}$$

Then \(u=v\), which is a contradiction. □

3 Particular cases

In this section, we deduce from our main theorems several fixed point theorems in b-metric spaces.

3.1 The class of ψ-type contractions in b-metric spaces

Definition 3.1

Let \((X,d)\) be a b-metric space with constant \(b\geq1\). A mapping \(T: X\to X\) is said to be a ψ-contraction if there exists \(\psi\in\Psi_{b}\) such that

$$ d(Tx,Ty)\leq\psi\bigl(d(x,y)\bigr)\quad \text{for all } x,y\in X. $$
(3.1)

Theorem 3.2

Let \((X,d)\) be a b-metric space with constant \(b\geq1\), and let \(T: X\to X\) be a given mapping. Suppose that there exists \(\psi\in\Psi_{b}\) such that T is a ψ-contraction. Then there exists \(\alpha: X\times X \to\mathbb{R}\) such that T is an α-ψ contraction.

Proof

Consider the function \(\alpha: X\times X \to\mathbb{R}\) defined by

$$ \alpha(x,y)=1\quad \text{for all } x,y\in X. $$
(3.2)

Clearly, from (3.1), T is an α-ψ contraction. □

Corollary 3.3

([17])

Let \((X,d)\) be a complete b-metric space with constant \(b\geq1\), and let \(T: X\to X\) be a given mapping. If T is a ψ-contraction for some \(\psi\in\Psi_{b}\), then T has a unique fixed point. Moreover, for any \(x_{0}\in X\), the Picard sequence \(\{T^{n}x_{0}\}\) converges to this fixed point.

Proof

From Lemma 1.6, we have

$$d(Tx,Ty)\leq d(x,y) \quad \text{for all } x,y\in X, $$

which implies that T is a continuous mapping. From Theorem 3.2, T is an α-ψ contraction, where α is defined by (3.2). Clearly, for any \(x_{0}\in X\), (2.1) is satisfied with \(p=1\) and \(\sigma=1\). By Theorem 2.2, \(\{T^{n}x_{0}\}\) converges to a fixed point of T. The uniqueness follows immediately from (3.2) and Theorem 2.4. □

Corollary 3.4

Let \((X,d)\) be a complete b-metric space with constant \(b\geq1\), and let \(T: X\to X\) be a given mapping. Suppose that

$$d(Tx,Ty)\leq k d(x,y)\quad \textit{for all } x,y\in X $$

for some constant \(k\in(0,1/b)\). Then T has a unique fixed point. Moreover, for any \(x_{0}\in X\), the Picard sequence \(\{T^{n}x_{0}\}\) converges to this fixed point.

Proof

It is an immediate consequence of Corollary 3.3 with \(\psi(t)=kt\). □

3.2 The class of rational-type contractions in b-metric spaces

3.2.1 Dass-Gupta-type contraction in b-metric spaces

Definition 3.5

Let \((X,d)\) be a b-metric space with constant \(b\geq1\). A mapping \(T: X\to X\) is said to be a Dass-Gupta contraction if there exist constants \(\lambda,\mu\geq0\) with \(\lambda b+\mu<1\) such that

$$ d(Tx,Ty)\leq\mu d(y,Ty) \frac{1+d(x,Tx)}{1+d(x,y)}+\lambda d(x,y)\quad \text{for all } x,y\in X. $$
(3.3)

Theorem 3.6

Let \((X,d)\) be a b-metric space with constant \(b\geq1\), and let \(T: X\to X\) be a given mapping. Suppose that T is a Dass-Gupta contraction. Then there exist \(\psi\in \Psi_{b}\) and \(\alpha: X\times X \to\mathbb{R}\) such that T is an α-ψ contraction.

Proof

From (3.3), for all \(x,y\in X\), we have

$$d(Tx,Ty)-\mu d(y,Ty) \frac{1+d(x,Tx)}{1+d(x,y)}\leq\lambda d(x,y), $$

which yields

$$ \biggl(1-\mu\frac{d(y,Ty)(1+d(x,Tx))}{(1+d(x,y))d(Tx,Ty)} \biggr)d(Tx,Ty) \leq\lambda d(x,y), \quad x,y\in X, Tx\neq Ty. $$
(3.4)

Consider the functions \(\psi:[0,\infty)\to[0,\infty)\) and \(\alpha:X\times X\to\mathbb{R}\) defined by

$$ \psi(t)=\lambda t,\quad t\geq0 $$
(3.5)

and

$$ \alpha(x,y)= \textstyle\begin{cases} 1- \mu\frac{d(y,Ty)(1+d(x,Tx))}{(1+d(x,y))d(Tx,Ty)}, & \text{if } Tx\neq Ty, \\ 0, & \text{otherwise}. \end{cases} $$
(3.6)

Since \(0\leq\lambda b<1\), then \(\psi\in\Psi_{b}\). On the other hand, from (3.4) we have

$$\alpha(x,y)d(Tx,Ty)\leq\psi\bigl(d(x,y)\bigr)\quad \text{for all } x,y\in X. $$

Then T is an α-ψ contraction. □

Corollary 3.7

Let \((X,d)\) be a complete b-metric space with constant \(b\geq1\), and let \(T: X\to X\) be a given mapping. If T is a Dass-Gupta contraction with parameters \(\lambda,\mu\geq0\) such that \(\lambda b+\mu<1\), then T has a unique fixed point. Moreover, for any \(x_{0}\in X\), the Picard sequence \(\{ T^{n}x_{0}\}\) converges to this fixed point.

Proof

Let \(x_{0}\) be an arbitrary point in X. If for some \(r\in\mathbb{N}\), \(T^{r}x_{0}=T^{r+1}x_{0}\), then \(T^{r}x_{0}\) will be a fixed point of T. So we can suppose that \(T^{r}x_{0}\neq T^{r+1}x_{0}\) for all \(r\in\mathbb{N}\). From (3.6), for all \(n\in\mathbb{N}\), we have

$$\begin{aligned} \alpha\bigl(T^{n}x_{0},T^{n+1}x_{0} \bigr) &= 1- \mu\frac{d(T^{n+1}x_{0},T^{n+2}x_{0}) (1+d(T^{n}x_{0},T^{n+1}x_{0}))}{(1+d(T^{n}x_{0},T^{n+1}x_{0}))d(T^{n+1}x_{0},T^{n+2}x_{0})} \\ &= 1-\mu>0. \end{aligned}$$

On the other hand, from (3.5) we have

$$(1-\mu)^{-1}\psi(t)=\frac{\lambda}{1-\mu}t, \quad t\geq0. $$

From the condition \(\lambda b+\mu<1\), clearly we have \((1-\mu)^{-1}\psi \in\Psi_{b}\), which is equivalent to \((1-\mu)^{-1}\in\Sigma_{\psi}^{b}\). Then (2.1) is satisfied with \(p=1\) and \(\sigma=(1-\mu)^{-1}\). From the first part of Theorem 2.3, the sequence \(\{T^{n}x_{0}\}\) converges to some \(x^{*}\in X\).

Suppose that \(x^{*}\) is not a fixed point of T, that is, \(d(x^{*},Tx^{*})>0\). Then

$$T^{n+1}x_{0}\neq Tx^{*},\quad n \mbox{ large enough}. $$

From (3.6), we have

$$\alpha\bigl(x^{*},T^{n}x_{0}\bigr) = 1-\mu\frac{d(T^{n}x_{0},T^{n+1}x_{0})(1+d(x^{*},Tx^{*}))}{(1+d(T^{n}x_{0},x^{*})) d(T^{n+1}x_{0},Tx^{*})}, \quad n \mbox{ large enough}. $$

On the other hand, using the property (iii) of a b-metric, we have

$$d\bigl(T^{n+1}x_{0},Tx^{*}\bigr)\geq\frac{1}{b}d \bigl(x^{*},Tx^{*}\bigr)-d\bigl(x^{*},T^{n+1}x_{0}\bigr)>0,\quad n \mbox{ large enough}. $$

Thus we have

$$\alpha\bigl(x^{*},T^{n}x_{0}\bigr)\geq1-\mu \frac{d(T^{n}x_{0},T^{n+1}x_{0})(1+d(x^{*},Tx^{*}))}{(1+d(T^{n}x_{0},x^{*})) (\frac{1}{b}d(x^{*},Tx^{*})-d(x^{*},T^{n+1}x_{0}) )}, \quad n \mbox{ large enough}. $$

Since

$$\lim_{n\to\infty} 1-\mu \frac{d(T^{n}x_{0},T^{n+1}x_{0})(1+d(x^{*},Tx^{*}))}{(1+d(T^{n}x_{0},x^{*})) (\frac{1}{b}d(x^{*},Tx^{*})-d(x^{*},T^{n+1}x_{0}) )}=1, $$

we have

$$\alpha\bigl(x^{*},T^{n}x_{0}\bigr)>\frac{1}{2}, \quad n \mbox{ large enough}. $$

By Theorem 2.3, we deduce that \(x^{*}\in \operatorname{Fix}(T)\), which is a contradiction. Thus \(\operatorname {Fix}(T)\neq\emptyset\).

For the uniqueness, observe that for every pair \((x,y)\in\operatorname {Fix}(T)\times\operatorname{Fix}(T)\) with \(x\neq y\), we have \(\alpha(x,y)=1\). By Theorem 2.4, \(x^{*}\) is the unique fixed point of T. □

If \(b=1\), Corollary 3.7 recovers the Dass-Gupta fixed point theorem [18].

3.2.2 Jaggi-type contraction in b-metric spaces

Definition 3.8

Let \((X,d)\) be a b-metric space with constant \(b\geq1\). A mapping \(T: X\to X\) is said to be a Jaggi contraction if there exist constants \(\lambda,\mu\geq0\) with \(\lambda b+\mu<1\) such that

$$ d(Tx,Ty)\leq\mu\frac{d(x,Tx)d(y,Ty)}{d(x,y)}+\lambda d(x,y)\quad \text{for all } x,y\in X, x\neq y. $$
(3.7)

Theorem 3.9

Let \((X,d)\) be a b-metric space with constant \(b\geq1\), and let \(T: X\to X\) be a given mapping. Suppose that T is a Jaggi contraction. Then there exist \(\psi\in\Psi_{b}\) and \(\alpha: X\times X \to\mathbb{R}\) such that T is an α-ψ contraction.

Proof

From (3.7), for all \(x,y\in X\) with \(x\neq y\), we have

$$d(Tx,Ty)-\mu\frac{d(x,Tx)d(y,Ty)}{d(x,y)}\leq\lambda d(x,y), $$

which yields

$$ \biggl(1-\mu\frac{d(x,Tx)d(y,Ty)}{d(x,y)d(Tx,Ty)} \biggr)d(Tx,Ty)\leq \lambda d(x,y), \quad x,y\in X, Tx\neq Ty. $$
(3.8)

Consider the functions \(\psi:[0,\infty)\to[0,\infty)\) and \(\alpha:X\times X\to\mathbb{R}\) defined by

$$ \psi(t)=\lambda t,\quad t\geq0 $$
(3.9)

and

$$ \alpha(x,y)= \textstyle\begin{cases} 1- \mu\frac{d(x,Tx)d(y,Ty)}{d(x,y)d(Tx,Ty)}, & \text{if } Tx\neq Ty, \\ 0, & \text{otherwise}. \end{cases} $$
(3.10)

Since \(\lambda b<1\), we have \(\psi\in\Psi_{b}\). From (3.8), we have

$$\alpha(x,y)d(Tx,Ty)\leq\psi\bigl(d(x,y)\bigr) \quad \text{for all } x,y\in X. $$

Then T is an α-ψ contraction. □

Corollary 3.10

Let \((X,d)\) be a complete b-metric space with constant \(b\geq1\), and let \(T: X\to X\) be a continuous mapping. If T is a Jaggi contraction with parameters \(\lambda,\mu\geq0\) such that \(\lambda b+\mu<1\), then T has a unique fixed point. Moreover, for any \(x_{0}\in X\), the Picard sequence \(\{T^{n}x_{0}\}\) converges to this fixed point.

Proof

Let \(x_{0}\) be an arbitrary point in X. Without loss of generality, we can suppose that \(T^{r}x_{0}\neq T^{r+1}x_{0}\) for all \(r\in\mathbb{N}\). From (3.10), for all \(n\in\mathbb{N}\), we have

$$\alpha\bigl(T^{n}x_{0},T^{n+1}x_{0} \bigr) = 1- \mu\frac {d(T^{n}x_{0},T^{n+1}x_{0})d(T^{n+1}x_{0},T^{n+2}x_{0})}{d(T^{n}x_{0},T^{n+1}x_{0}) d(T^{n+1}x_{0},T^{n+2}x_{0})} = 1-\mu>0. $$

On the other hand, from (3.9), for all \(t\geq0\), we have

$$(1-\mu)^{-1}\psi(t)=\frac{\lambda}{1-\mu} t. $$

Since \(\lambda b+\mu<1\), we have \((1-\mu)^{-1}\psi\in\Psi_{b}\), that is, \((1-\mu)^{-1}\in\Sigma_{\psi}^{b}\). Then (2.1) is satisfied with \(p=1\) and \(\sigma=(1-\mu)^{-1}\). By the first part of Theorem 2.2, \(\{T^{n}x_{0}\}\) converges to some \(x^{*}\in X\). Since T is continuous, by the second part of Theorem 2.2, \(x^{*}\) is a fixed point of T. Moreover, for every pair \((x,y)\in\operatorname{Fix}(T)\times\operatorname{Fix}(T)\) with \(x\neq y\), we have \(\alpha(x,y)=1\). Then, by Theorem 2.4, \(x^{*}\) is the unique fixed point of T. □

If \(b=1\), Corollary 3.10 recovers the Jaggi fixed point theorem [19].

3.3 The class of Berinde-type mappings in b-metric spaces

Definition 3.11

Let \((X,d)\) be a b-metric space with constant \(b\geq1\). A mapping \(T: X\to X\) is said to be a Berinde-type contraction if there exist \(\lambda\in(0,1/b)\) and \(L\geq0\) such that

$$ d(Tx,Ty)\leq\lambda d(x,y)+L d(y,Tx) \quad \text{for all } x,y\in X. $$
(3.11)

Theorem 3.12

Let \((X,d)\) be a b-metric space with constant \(b\geq1\), and let \(T: X\to X\) be a given mapping. If T is a Berinde-type contraction, then there exist \(\alpha: X\times X \to\mathbb{R}\) and \(\psi\in\Psi_{b}\) such that T is an α-ψ contraction.

Proof

From (3.11), we have

$$d(Tx,Ty)-L d(y,Tx) \leq\lambda d(x,y) \quad \text{for all } x,y\in X, $$

which yields

$$ \biggl(1-L \frac{d(y,Tx)}{d(Tx,Ty)} \biggr)d(Tx,Ty)\leq\lambda d(x,y), \quad x,y\in X, Tx\neq Ty. $$
(3.12)

Consider the functions \(\psi:[0,\infty)\to[0,\infty)\) and \(\alpha:X\times X\to\mathbb{R}\) defined by

$$\psi(t)=\lambda t, \quad t\geq0 $$

and

$$ \alpha(x,y)= \textstyle\begin{cases} 1- L \frac{d(y,Tx)}{d(Tx,Ty)}, &\text{if } Tx\neq Ty, \\ 0, &\text{otherwise}. \end{cases} $$
(3.13)

Since \(\lambda b<1\), then \(\psi\in\Psi_{b}\). From (3.12), we have

$$\alpha(x,y)d(Tx,Ty)\leq\psi\bigl(d(x,y)\bigr)\quad \text{for all } x,y\in X. $$

Then T is an α-ψ contraction. □

Corollary 3.13

Let \((X,d)\) be a complete b-metric space with constant \(b\geq1\), and let \(T: X\to X\) be a given mapping. If T is a Berinde-type contraction with parameters \(\lambda,L \geq0\) such that \(0<\lambda b<1\), then for any \(x_{0}\in X\), the Picard sequence \(\{T^{n}x_{0}\}\) converges to a fixed point of T.

Proof

Let \(x_{0}\) be an arbitrary point in X. Without loss of generality, we can suppose that \(T^{r}x_{0}\neq T^{r+1}x_{0}\) for all \(r\in\mathbb{N}\). From (3.13), for all \(n\in\mathbb{N}\), we have

$$\alpha\bigl(T^{n}x_{0},T^{n+1}x_{0} \bigr) = 1- L \frac{d(T^{n+1}x_{0},T^{n+1}x_{0})}{d(T^{n+1}x_{0},T^{n+2}x_{0})} = 1. $$

Then (2.1) holds with \(\sigma=1\) and \(p=1\). From the first part of Theorem 2.3, the sequence \(\{T^{n}x_{0}\}\) converges to some \(x^{*}\in X\).

Suppose now that \(x^{*}\) is not a fixed point of T, that is, \(d(x^{*},Tx^{*})>0\). Then

$$T^{n+1}x_{0}\neq Tx^{*}, \quad n \mbox{ large enough}. $$

From (3.13), we have

$$\alpha\bigl(T^{n}x_{0},x^{*}\bigr)= 1-L \frac{d(x^{*},T^{n+1}x_{0})}{d(T^{n+1}x_{0},Tx^{*})}, \quad n \mbox{ large enough}. $$

Using the property (iii) of a b-metric, we have

$$d\bigl(T^{n+1}x_{0},Tx^{*}\bigr)\geq\frac{1}{b}d \bigl(x^{*},Tx^{*}\bigr)-d\bigl(x^{*},T^{n+1}x_{0}\bigr)>0,\quad n \mbox{ large enough}. $$

Thus we have

$$\alpha\bigl(T^{n}x_{0},x^{*}\bigr)\geq1 -L\frac{d(x^{*},T^{n+1}x_{0})}{\frac {1}{b}d(x^{*},Tx^{*})-d(x^{*},T^{n+1}x_{0})}, \quad n \mbox{ large enough}. $$

Since

$$\lim_{n\to\infty}1 -L\frac{d(x^{*},T^{n+1}x_{0})}{\frac {1}{b}d(x^{*},Tx^{*})-d(x^{*},T^{n+1}x_{0})}=1, $$

then

$$\alpha\bigl(T^{n}x_{0},x^{*}\bigr)>\frac{1}{2}, \quad n \mbox{ large enough}. $$

By Theorem 2.3, we deduce that \(x^{*}\in \operatorname{Fix}(T)\), which is a contradiction.

Thus \(x^{*}\) is a fixed point of T. □

If \(b=1\), Corollary 3.13 recovers the Berinde fixed point theorem [20].

Note that a Berinde mapping need not have a unique fixed point (see [21], Example 2.11).

Corollary 3.14

Let \((X,d)\) be a complete b-metric space with constant \(b\geq1\), and let \(T: X\to X\) be a given mapping. Suppose that there exists a constant \(k\in(0,1/b(b+1))\) such that

$$ d(Tx,Ty)\leq k \bigl(d(x,Tx)+d(y,Ty)\bigr) \quad \textit{for all } x,y\in X. $$
(3.14)

Then, for any \(x_{0}\in X\), the Picard sequence \(\{T^{n}x_{0}\}\) converges to a fixed point of T.

Proof

At first, observe that from (3.14), for all \(x,y\in X\), we have

$$d(Tx,Ty)\leq\lambda d(x,y)+L d(y,Tx), $$

where

$$\lambda=\frac{kb}{1-kb} \quad \mbox{and} \quad L=\frac{2kb}{1-kb}\cdot $$

With the condition \(k\in(0,1/b(b+1))\), we have \(0<\lambda<1/b\) and \(L\geq0\). Then T is a Berinde-type contraction. From Corollary 3.13, if \(x_{0}\in X\), then \(\{T^{n}x_{0}\}\) converges to a fixed point of T. □

If \(b=1\), Corollary 3.14 recovers the Kannan fixed point theorem [22].

Corollary 3.15

Let \((X,d)\) be a complete b-metric space with constant \(b\geq1\), and let \(T: X\to X\) be a given mapping. Suppose that there exists a constant \(k\in(0,1/2b^{2})\) such that

$$ d(Tx,Ty)\leq k \bigl(d(x,Ty)+d(y,Tx)\bigr) \quad \textit{for all } x,y\in X. $$
(3.15)

Then, for any \(x_{0}\in X\), the Picard sequence \(\{T^{n}x_{0}\}\) converges to a fixed point of T.

Proof

From (3.15), we have

$$d(Tx,Ty)\leq\lambda d(x,y)+L d(y,Tx), $$

where

$$\lambda=\frac{kb}{1-kb^{2}}\quad \mbox{and}\quad L=\frac {k(b^{2}+1)}{1-kb^{2}}\cdot $$

With the condition \(k\in(0,1/2b^{2})\), we have \(0<\lambda<1/b\) and \(L\geq0\). Then T is a Berinde-type contraction. From Corollary 3.13, if \(x_{0}\in X\), then \(\{T^{n}x_{0}\}\) converges to a fixed point of T. □

If \(b=1\), Corollary 3.15 recovers the Chatterjee fixed point theorem [23].

3.4 Ćirić-type mappings in b-metric spaces

Definition 3.16

Let \((X,d)\) be a b-metric space with constant \(b\geq1\). A mapping \(T: X\to X\) is said to be a Ćirić-type mapping if there exists \(\lambda\in(0,1/b)\) such that for all \(x,y\in X\), we have

$$ \min\bigl\{ d(Tx,Ty),d(x,Tx),d(y,Ty)\bigr\} -\min\bigl\{ d(x,Ty),d(y,Tx)\bigr\} \leq\lambda d(x,y). $$
(3.16)

Theorem 3.17

Let \((X,d)\) be a b-metric space with constant \(b\geq1\), and let \(T: X\to X\) be a given mapping. If T is a Ćirić-type mapping with parameter \(\lambda\in(0,1/b)\), then there exist \(\alpha: X\times X \to\mathbb{R}\) and \(\psi\in\Psi _{b}\) such that T is an α-ψ contraction.

Proof

Consider the functions \(\psi:[0,\infty)\to[0,\infty)\) and \(\alpha:X\times X\to\mathbb{R}\) defined by

$$ \psi(t)=\lambda t,\quad t\geq0 $$
(3.17)

and

$$ \alpha(x,y)= \textstyle\begin{cases} \min \{1,\frac{d(x,Tx)}{d(Tx,Ty)},\frac{d(y,Ty)}{d(Tx,Ty)} \} -\min \{\frac{d(x,Ty)}{d(Tx,Ty)},\frac{d(y,Tx)}{d(Tx,Ty)} \} , &\text{if } Tx\neq Ty, \\ 0, &\text{otherwise}. \end{cases} $$
(3.18)

From (3.16), we have

$$ \alpha(x,y)d(Tx,Ty)\leq\psi\bigl(d(x,y)\bigr)\quad \text{for all } x,y\in X, $$
(3.19)

which implies that T is an α-ψ contraction. □

Corollary 3.18

Let \((X,d)\) be a complete b-metric space with constant \(b\geq1\), and let \(T: X\to X\) be a continuous mapping. If T is a Ćirić-type mapping with parameter \(\lambda\in(0,1/b)\), then for any \(x_{0}\in X\), the Picard sequence \(\{T^{n}x_{0}\}\) converges to a fixed point of T.

Proof

Let \(x_{0}\in X\) be an arbitrary point. Without loss of generality, we can suppose that \(T^{r}x_{0}\neq T^{r+1}x_{0}\) for all \(r\in\mathbb{N}\). From (3.18), for all \(n\in\mathbb{N}\), we have

$$\begin{aligned} \alpha\bigl(T^{n}x_{0},T^{n+1}x_{0} \bigr) =& \min \biggl\{ 1,\frac{d(T^{n}x_{0},T^{n+1}x_{0})}{d(T^{n+1}x_{0},T^{n+2}x_{0})}, \frac{d(T^{n+1}x_{0},T^{n+2}x_{0})}{d(T^{n+1}x_{0},T^{n+2}x_{0})} \biggr\} \\ &{} -\min \biggl\{ \frac{d(T^{n}x_{0},T^{n+2}x_{0})}{d(T^{n+1}x_{0},T^{n+2}x_{0})}, \frac{d(T^{n+1}x_{0},T^{n+1}x_{0})}{d(T^{n+1}x_{0},T^{n+2}x_{0})} \biggr\} \\ =& \min \biggl\{ 1,\frac{d(T^{n}x_{0},T^{n+1}x_{0})}{d(T^{n+1}x_{0},T^{n+2}x_{0})} \biggr\} . \end{aligned}$$

Suppose that for some \(n\in\mathbb{N}\), we have

$$\alpha\bigl(T^{n}x_{0},T^{n+1}x_{0} \bigr)=\frac{d(T^{n}x_{0},T^{n+1}x_{0})}{d(T^{n+1}x_{0},T^{n+2}x_{0})}. $$

In this case, from (3.17) and (3.19), we have

$$d\bigl(T^{n}x_{0},T^{n+1}x_{0}\bigr) \leq\lambda d\bigl(T^{n}x_{0},T^{n+1}x_{0} \bigr). $$

This implies (from the assumption \(T^{r}x_{0}\neq T^{r+1}x_{0}\) for all \(r\in\mathbb{N}\)) that \(\lambda\geq1\), which is a contradiction. Then

$$\alpha\bigl(T^{n}x_{0},T^{n+1}x_{0} \bigr)=1 \quad \text{for all } n\in\mathbb{N}. $$

Then (2.1) is satisfied with \(p=1\) and \(\sigma=1\). By Theorem 2.3, we deduce that the sequence \(\{T^{n}x_{0}\}\) converges to a fixed point of T. □

If \(b=1\), Corollary 3.18 recovers Ćirić’s fixed point theorem [24].

3.5 Edelstein fixed point theorem in b-metric spaces

Another consequence of our main results is the following generalized version of Edelstein fixed point theorem [25] in b-metric spaces.

Corollary 3.19

Let \((X,d)\) be a complete b-metric space with constant \(b\geq1\), and ε-chainable for some \(\varepsilon>0\); i.e., given \(x,y\in X\), there exist a positive integer N and a sequence \(\{x_{i}\}_{i=0}^{N}\subset X\) such that

$$ x_{0}=x,\qquad x_{N}=y,\qquad d(x_{i},x_{i+1})< \varepsilon \quad \textit{for } i=0, \ldots,N-1. $$
(3.20)

Let \(T: X\to X\) be a given mapping such that

$$ x,y\in X, \quad d(x,y)< \varepsilon\quad \Longrightarrow \quad d(Tx,Ty)\leq\psi\bigl(d(x,y)\bigr) $$
(3.21)

for some \(\psi\in\Psi_{b}\). Then T has a unique fixed point.

Proof

It is clear from (3.21) that the mapping T is continuous. Now, consider the function \(\alpha:X\times X\to\mathbb{R}\) defined by

$$ \alpha(x,y)= \textstyle\begin{cases} 1, &\text{if } d(x,y)< \varepsilon, \\ 0, &\text{otherwise}. \end{cases} $$
(3.22)

From (3.21), we have

$$\alpha(x,y)d(Tx,Ty)\leq\psi\bigl(d(x,y)\bigr) \quad \text{for all } x,y\in X. $$

Let \(x_{0}\in X\). For \(x=x_{0}\) and \(y=Tx_{0}\), from (3.20) and (3.22), for some positive integer p, there exists a finite sequence \(\{\xi_{i}\}_{i=0}^{p}\subset X\) such that

$$x_{0}=\xi_{0}, \qquad \xi_{p}=Tx_{0}, \qquad \alpha(\xi_{i},\xi_{i+1})\geq1\quad \text{for } i=0, \ldots,p-1. $$

Now, let \(i\in\{0,\ldots,p-1\}\) be fixed. From (3.22) and (3.21), we have

$$\begin{aligned} \alpha(\xi_{i},\xi_{i+1})\geq1 &\quad \Longrightarrow\quad d(\xi_{i},\xi_{i+1})< \varepsilon \\ &\quad \Longrightarrow\quad d(T\xi_{i},T\xi_{i+1})\leq\psi \bigl(d(\xi_{i},\xi_{i+1})\bigr) \leq d(\xi_{i}, \xi_{i+1})< \varepsilon \\ &\quad \Longrightarrow\quad \alpha(T\xi_{i},T\xi_{i+1}) \geq1. \end{aligned}$$

Again,

$$\begin{aligned} \alpha(T\xi_{i},T\xi_{i+1})\geq1 &\quad \Longrightarrow\quad d(T\xi_{i},T\xi_{i+1})< \varepsilon \\ &\quad \Longrightarrow\quad d\bigl(T^{2}\xi_{i},T^{2} \xi_{i+1}\bigr)\leq\psi\bigl(d(T\xi_{i},T\xi _{i+1}) \bigr)\leq d(T\xi_{i},T\xi_{i+1})< \varepsilon \\ &\quad \Longrightarrow \quad \alpha\bigl(T^{2}\xi_{i},T^{2} \xi_{i+1}\bigr)\geq1. \end{aligned}$$

By induction, we obtain

$$\alpha\bigl(T^{n}\xi_{i},T^{n+1}\xi_{i+1} \bigr)\geq1 \quad \text{for all } n\in\mathbb{N}. $$

Then (2.1) is satisfied with \(\sigma=1\). From Theorem 2.2, the sequence \(\{T^{n}x_{0}\}\) converges to a fixed point of T. Using a similar argument, we can see that condition (ii) of Theorem 2.4 is satisfied, which implies that T has a unique fixed point. □

3.6 Contractive mapping theorems in b-metric spaces with a partial order

Let \((X,d)\) be a b-metric space with constant \(b\geq1\), and let ⪯ be a partial order on X. We denote

$$\Delta=\bigl\{ (x,y)\in X\times X: x\preceq y \text{ or } y\preceq x\bigr\} . $$

Corollary 3.20

Let \(T: X\to X\) be a given mapping. Suppose that there exists \(\psi\in\Psi_{b}\) such that

$$ d(Tx,Ty)\leq\psi\bigl(d(x,y)\bigr) \quad \textit{for all } (x,y) \in\Delta. $$
(3.23)

Suppose also that

  1. (i)

    T is continuous;

  2. (ii)

    for some positive integer p, there exists a finite sequence \(\{\xi_{i}\}_{i=0}^{p}\subset X\) such that

    $$ \xi_{0}=x_{0},\qquad \xi_{p}=Tx_{0}, \qquad \bigl(T^{n}\xi_{i},T^{n}\xi_{i+1} \bigr)\in\Delta, \quad n\in\mathbb{N}, i=0,\ldots,p-1. $$
    (3.24)

Then \(\{T^{n}x_{0}\}\) converges to a fixed point of T.

Proof

Consider the function \(\alpha:X\times X\to\mathbb {R}\) defined by

$$ \alpha(x,y)= \textstyle\begin{cases} 1, &\text{if } (x,y)\in\Delta, \\ 0, &\text{otherwise}. \end{cases} $$
(3.25)

From (3.23), we have

$$\alpha(x,y)d(Tx,Ty)\leq\psi\bigl(d(x,y)\bigr) \quad \text{for all } x,y\in X. $$

Then the result follows from Theorem 2.2 with \(\sigma=1\). □

Corollary 3.21

Let \(T: X\to X\) be a given mapping. Suppose that

  1. (i)

    there exists \(\psi\in\Psi_{b}\) such that (3.23) holds;

  2. (ii)

    condition (3.24) holds.

Then \(\{T^{n}x_{0}\}\) converges to some \(x^{*}\in X\). Moreover, if

  1. (iii)

    there exist a subsequence \(\{T^{\gamma(n)}x_{0}\}\) of \(\{ T^{n}x_{0}\}\) and \(N\in\mathbb{N}\) such that

    $$\bigl(T^{\gamma(n)}x_{0},x^{*}\bigr)\in\Delta, \quad n\geq N, $$

then \(x^{*}\) is a fixed point of T.

Proof

We continue to use the same function α defined by (3.25). From the first part of Theorem 2.3, the sequence \(\{T^{n}x_{0}\}\) converges to some \(x^{*}\in X\). From (iii) and (3.25), we have

$$\alpha\bigl(T^{\gamma(n)}x_{0},x^{*}\bigr)=1, \quad n\geq N. $$

By the second part of Theorem 2.3 (with \(\ell=1\)), we deduce that \(x^{*}\) is a fixed point of T. □

The next result follows from Theorem 2.4 with \(\eta=1\).

Corollary 3.22

Let \(T: X\to X\) be a given mapping. Suppose that

  1. (i)

    there exists \(\psi\in\Psi_{b}\) such that (3.23) holds;

  2. (ii)

    \(\operatorname{Fix}(T)\neq\emptyset\);

  3. (iii)

    for every pair \((x,y)\in\operatorname{Fix}(T) \times\operatorname{Fix}(T)\) with \(x\neq y\), if \((x,y)\notin\Delta\), there exist a positive integer q and a finite sequence \(\{\zeta_{i}(x,y)\}_{i=0}^{q}\subset X\) such that

    $$\zeta_{0}(x,y)=x,\qquad \zeta_{q}(x,y)=y,\qquad \bigl(T^{n}\zeta_{i}(x,y),T^{n} \zeta_{i+1}(x,y)\bigr)\in\Delta $$

    for \(n\in\mathbb{N}\) and \(i=0,\ldots,q-1\).

Then T has a unique fixed point.

Observe that in our results we do not suppose that T is monotone or T preserves order as it is supposed in many papers (see [2628] and others).